Difference between revisions of "2006 AMC 10A Problems/Problem 4"

 
(Redirect)
 
(5 intermediate revisions by 4 users not shown)
Line 1: Line 1:
== Problem ==
+
#redirect [[2006 AMC 12A Problems/Problem 4]]
A digital watch displays hours and minutes with AM and PM.  What is the largest possible sum of the digits in the display?
 
 
 
<math> \mathrm{(A) \ } 17\qquad \mathrm{(B) \ } 19\qquad \mathrm{(C) \ } 21\qquad \mathrm{(D) \ } 22\qquad \mathrm{(E) \ } 23 </math>
 
== Solution ==
 
== See Also ==
 
*[[2006 AMC 10A Problems]]
 

Latest revision as of 00:05, 28 April 2008